Question

A series RCL circuit has a resonant frequency of 1200 Hz. When operating at a frequency...

A series RCL circuit has a resonant frequency of 1200 Hz. When operating at a frequency other than 1200 Hz, the circuit has a capacitive reactance of 7.00 and an inductive reactance of 41.0. What are the values of (a) L and (b) C? Note: The ac current and voltage are rms values and power is an average value unless indicated otherwise.

Homework Answers

Answer #1

As we know that the impedence of series RLC circuit is given by -

Z= sqrt[R^2 + (Lw - 1/Cw)^2]

Now at resonance -

w=wo, (wo=2*pi*fo & fo = 1200 Hz (resonant freq.))

Z has to be the least (Z=R) or current the MAXIMUM

Lwo = (1/Cwo) or wo^2 = 1/LC or

LC = 1 / 4*pi^2*fo^2 ---------------------------------------------(i)

When this circuit operates on any other frequency (v), then

Inductive reactance= L.(2*pi*v) = 41--------------------------------(ii)
Capacitive reactance= 1 / (C*2*pi*v) = 7----------------------------(iii)

Now, multiply (ii) and (iii), we get -

L / C = 287---------------------------------(iv)

Multiply (i) and (iv)

L^2 = [1 / 4*pi^2*fo^2] * 287

=> L = sqrt(287)/2*pi*fo
L = 16.9 / (2*3.14*1200) = 0.002243 = 2243 x 10^-6 H = 2243 micro H

And -

C = L /150 = 2243 / 150 = 14.95 micro farad.

(a) L = 2243 H

(b) C = 14.95 F

Know the answer?
Your Answer:

Post as a guest

Your Name:

What's your source?

Earn Coins

Coins can be redeemed for fabulous gifts.

Not the answer you're looking for?
Ask your own homework help question
Similar Questions
A series RCL circuit has a resonant frequency of 1900 Hz. When operating at a frequency...
A series RCL circuit has a resonant frequency of 1900 Hz. When operating at a frequency other than 1900 Hz, the circuit has a capacitive reactance of 3.90 and an inductive reactance of 40.0. What are the values of (a) L and (b) C? Note: The ac current and voltage are rms values and power is an average value unless indicated otherwise.
A series RCL circuit contains only a capacitor (C = 12.2 μF), an inductor (L =...
A series RCL circuit contains only a capacitor (C = 12.2 μF), an inductor (L = 5.28 mH), and a generator (peak voltage = 68.5 V, frequency = 5.60 x 103 Hz). When t = 0 s, the instantaneous value of the voltage is zero, and it rises to a maximum one-quarter of a period later. (a) Find the instantaneous value of the voltage across the capacitor/inductor combination when t = 7.20 x 10-4 s. (b) What is the instantaneous...
1.The resonant frequency of an RCL circuit is 3.0 kHz, and the value of the inductance...
1.The resonant frequency of an RCL circuit is 3.0 kHz, and the value of the inductance is 5.0 mH. What is the resonant frequency (in kHz) when the value of the inductance is 1.5 mH? 2. Part a of the drawing shows a resistor and a charged capacitor wired in series. When the switch is closed, the capacitor discharges as charge moves from one plate to the other. Part b shows the amount q of charge remaining on each plate...
An RLC circuit consists of an alternating voltage source with RMS voltage 12 V and frequency...
An RLC circuit consists of an alternating voltage source with RMS voltage 12 V and frequency 70 Hz, a 200 Ohm resistor, a 1.2 H inductor, and an 800 nF capacitor, all wired in series. a)  What is the inductive reactance of the circuit? b)  What is the capacitive reactance of the circuit? c)  What is the impedance of the circuit? d)  What is the RMS current in the circuit? e)  If the frequency is adjustable, what frequency should you use to maximize the current...
The RLC series circuit illustrated in the simulation has R = 1.49 Ω, L = 1.25...
The RLC series circuit illustrated in the simulation has R = 1.49 Ω, L = 1.25 H, and C = 198 µF. The applied AC voltage has a frequency of f = 60 Hz and a voltage of Δv = 120 V. 1-Find the inductive reactance, capacitive reactance, and impedance. XL = ........ Ω XC = ........ Ω Z = ......... Ω 2-Find the phase difference between current and voltage............. ° 3-Find the voltages ΔvR, ΔvL, and ΔvC. ΔvR =...
An AC voltage source has an RMS voltage of 10 volts and it is connected in...
An AC voltage source has an RMS voltage of 10 volts and it is connected in series to an inductor, a capacitor, and a resistor. At the resonant frequency the maximum current in the circuit is 7 amps and the inductive reactance is 10 Ohms. What is the maximum current in the circuit, in amps, at a frequency which is a factor of 2.5 more than the resonant frequency? ***PLEASE ONLY ANSWER IF 100% CORRECT***
Now Tiva and Graciela try a problem from the book. The RLC series circuit illustrated in...
Now Tiva and Graciela try a problem from the book. The RLC series circuit illustrated in the simulation has R = 1.2 Ω, L = 1.02 H, and C = 183 µF. The applied AC voltage has a frequency of f = 60 Hz and a voltage of Δv = 120 V. Find the inductive reactance, capacitive reactance, and impedance. XL =____ Ω XC =____ Ω Z =____ Ω Find the phase difference between current and voltage. ° Find the...
A series R-L-C circuit has a series resonant frequency of 12,000 Hz. a. If R =...
A series R-L-C circuit has a series resonant frequency of 12,000 Hz. a. If R = 5 ?, and if XL at resonance is 300 ?, find the bandwidth. b. Find the cutoff frequencies
A series AC circuit contains a resistor, an inductor of 210 mH, a capacitor of 5.50...
A series AC circuit contains a resistor, an inductor of 210 mH, a capacitor of 5.50 µF, and a source with ΔVmax = 240 V operating at 50.0 Hz. The maximum current in the circuit is 200 mA. (a) Calculate the inductive reactance. Ω (b) Calculate the capacitive reactance. Ω (c) Calculate the impedance. kΩ (d) Calculate the resistance in the circuit. kΩ (e) Calculate the phase angle between the current and the source voltage. °
A generator with rms voltage of rms = 120 V drives an RLC circuit at frequency...
A generator with rms voltage of rms = 120 V drives an RLC circuit at frequency f = 60 Hz. The load resistance R and reactance values of the inductor L and the capacitor C of the circuit are given by R = 50 , XL = 50 , XC = 150 , respectively What is the impedance of the circuit? What is the peak current amplitude in the circuit? What is the phase angle  of the circuit? The...